Thứ Tư, 26 tháng 10, 2016

Làm giảm biến số trong bất đẳng thức

Đề bài: Cho $a,b,c \ge 0,a+b+c=2$, Chứng minh rằng
$$\frac{1}{1+a^2+b^2}+\frac{1}{1+b^2+c^2}+\frac{1}{1+c^2+a^2} \ge \frac{4}{3}$$

Lời giải:
Ta thấy dấu bằng xảy ra khi a=b=1, c=0 và các hoán vị, vì thế ta sẽ đánh giá cho đúng dấu bằng xảy ra
Cách 1: Không mất tính tổng quát giả sử $ a \ge b \ge c \ge 0$

Ta có
$a^2+b^2\leq (a+\frac{c}{2})^2+(b+\frac{c}{2})^2$
$b^2+c^2\leq b^2+bc+\frac{c^2}{4}=(b+\frac{c}{2})^2$
$a^2+c^2\leq a^2+ac+\frac{c^2}{4}=(a+\frac{c}{2})^2$
Đặt $b+\frac{c}{2}=x,a+\frac{c}{2}=y$ thì $x+y=2$ Và ta chỉ cần chứng minh
\[\frac{1}{1+x^2+y^2}+\frac{1}{1+x^2}+\frac{1}{1+y^2}\geq \frac{4}{3}\]
\[\Leftrightarrow 5+4(x^2+y^2)\geq x^4+y^4+3x^2y^2+4x^2y^2(x^2+y^2)\]
Đặt $x^2+y^2=A,x^2y^2=B$ Thì bất đẳng thức trở thành
$A^2+B+4AB\leq 5+4A$ với $A+2\sqrt{B}=4\Leftrightarrow (4-A)^2=4B$
Như vậy  $(A-2)(4A^2-19A+2)\leq 0$ Đúng vì $2\leq A\leq 4$.

Cách 2:
Đặt $p = a+b+c=2,\,q=ab+bc+ca,r=abc$ bất đẳng thước tương đương với
\[4r^2+4(17-8q)r+(1-q)(5+13q-8q^2) \ge 0. \quad (1)\]
Nếu $q \leqslant 1$  $(1)$ đúng
Nếu $q \geqslant 1$ Dùng bất đẳng thức Schur $r \geqslant \frac{8}{9}q-\frac{8}{9}$ Ta cần chỉ ra
\[4\left(\frac{8}{9}q-\frac{8}{9}\right)^2+4(17-8q)\left(\frac{8}{9}q-\frac{8}{9}\right)+(1-q)(5+13q-8q^2) \ge 0,\]
Tương đương
\[\frac{1}{81}(q-1)(648q^2-3101q+4235) \geqslant 0.\]
Rõ ràng đúng.



Thứ Ba, 25 tháng 10, 2016

Dồn biến trong bất đẳng thức

Bài toán 1: Cho $ a, b, c > 0$  và $ abc = 1$. Chứng minh rằng: $ 81(1 + a^2)(1 + b^2)(1 + c^2) \leq 8(a+b+c)^4$

Lời giải:

Cách 1:

Đặt $ f(a,b,c)=8(a+b+c)^4-81(1+a^2)(1+b^2)(1+c^2)$ và $ \frac{4a}{(\sqrt b+\sqrt c)^2}=t.$
Vì thế, $ f(a,b,c)-f(a,\sqrt{bc},\sqrt{bc})=$
$ =8(\sqrt b-\sqrt c)^2(2a+b+c+2\sqrt{bc})((a+b+c)^2+(a+2\sqrt{bc})^2)-$
$ -81(\sqrt b-\sqrt c)^2(1+a^2)(\sqrt b+\sqrt c)^2)=$
$ =8(\sqrt b-\sqrt c)^2(2a+(\sqrt b+\sqrt c)^2)(2a^2+2a(\sqrt b+\sqrt c)^2+b^2+6bc+c^2)-$
$ -81(\sqrt b-\sqrt c)^2(\sqrt[3]{a^2b^2c^2}+a^2)(\sqrt b+\sqrt c)^2)\geq$
$ \geq8(\sqrt b-\sqrt c)^2(2a+(\sqrt b+\sqrt c)^2)(2a^2+2a(\sqrt b+\sqrt c)^2+\frac{(\sqrt b+\sqrt c)^4}{2})-$
$ -81(\sqrt b-\sqrt c)^2(\sqrt[3]{a^2\cdot\frac{(\sqrt b+\sqrt c)^8}{256}}+a^2)(\sqrt b+\sqrt c)^2)=$
$ =\frac{(\sqrt b-\sqrt c)^2(\sqrt b+\sqrt c)^6}{16}(8(t+2)^3-81(\sqrt[3]{t^2}+t^2))\geq0$ since,
$ 8(t+2)^3-81(\sqrt[3]{t^2}+t^2)=8t^3-33t^2+96t-81\sqrt[3]{t^2}+64=$
$ =(8t^3-33t^2+42t+37)+(54t-81\sqrt[3]{t^2}+27)\geq0.$
Ta chỉ cần chứng minh rằng:
 $ f(\frac{1}{b^2},b,b)\geq0.$
Nhưng $ f(\frac{1}{b^2},b,b)\geq0\Leftrightarrow\frac{(1+2b^3)^4}{(1+b^4)(1+b^2)^2b^4}\geq\frac{81}{8}.$
Đặt $ g(b)=\ln{\frac{(1+2b^3)^4}{(1+b^4)(1+b^2)^2b^4}}.$
Vì thế, $ g'(b)=\frac{24b^2}{1+2b^3}-\frac{4b^3}{1+b^4}-\frac{4b}{1+b^2}-\frac{4}{b}=$
$ =\frac{4(b-1)(2b^8+2b^6+2b^4-b^3+3b^2+b+1)}{b(1+2b^3)(1+b^2)(1+b^4)}.$
Nên $ b_{min}=1$ Và $ \min_{b>0}g=g(1)=\ln\frac{81}{8}$ ta có đpcm

Chú ý ở đây việc lấy ln đã làm việc xét đạo hàm gọn hơn rất nhiều

Cách 2:

Đặt $a+b+c=p,ab+bc+ca=q$, Bất đẳng thức trở thành:
$$p^4-6p^2+24p-12q^2+12q-27 \ge0$$
Đây là một hàm bậc hai theo q.
Vì $q\ge 3>1/2$, Ta chỉ cần chứng minh khi q đạt giá trị lớn nhất
Áp dụng bdt Schur $q\le \frac{p^3+9}{4p}$ Vậy
$$p^4-6p^2+24p-27-\frac{3(p^6+18p^3+81)}{4p^2}+\frac{3(p^3+9)}{p} \ge0$$
$$(p-3)(p^5+3p^4-3p^3+33p^2-9p+81)\ge0$$
Luôn đúng do $p\ge 3$.

Bài toán 2:

cho a,b,c là các số thực dương thỏa mãn abc=1 chứng minh rằng:

$\frac{a}{(a+3)^{2}}+\frac{b}{(b+3)^{2}}+\frac{c}{(c+3)^{2}}\leq \frac{3}{16}$

Lời giải:

Đặt $a=\frac1x, b=\frac1y, c=\frac1z \implies xyz=1$
Bất đẳng thức trở thành
\[\frac{x}{(3x+1)^2}+\frac{y}{(3y+1)^2}+\frac{z}{(3z+1)^2}\le \frac3{16}\]
Tương đương
\[\sum_{cyc}\left(\frac1{12}-\frac{x}{(3x+1)^2}\right) \ge \frac1{16} \iff \sum_{cyc}\frac{(3x-1)^2}{(3x+1)^2} \ge \frac34\]
Áp dụng bất đẳng thức C-S
\[\sum_{cyc}\frac{(3x-1)^2}{(3x+1)^2} \ge \frac{9(x+y+z-1)^2}{\sum (3x-1)^2}\]
Ta chỉ cần chứng minh:
\[f(x,y,z)=3+\sum x^2+8\sum xy-10\sum x \ge 0\]
Không mất tính tổng quát giả sử $x=\max(x,y,z) \implies x \ge 1$.
\[f(x,y,z)-f(x,\sqrt{yz},\sqrt{yz})=(\sqrt{y}-\sqrt{z})^2((\sqrt{y}+\sqrt{z})^2+8x-10) \ge 0\]
Do \[(\sqrt{y}+\sqrt{z})^2+8x \ge 4\sqrt{yz}+4x+4x \ge 8\sqrt{x\sqrt{yz}}+4x=8\sqrt[4]{x}+4x \ge 12\]
Cần chứng minh: \[f\left(\frac1{t^2},t,t\right) \ge 0\]
Tức là
\[((3t^2-1)^2+t(1-t)+t^4+t)(t-1)^2 \ge 0\]
Ta có điều phải chứng minh. Dấu bằng xảy ra khi $a=b=c=1$.

Các bạn thử bài này dùng bằng bất đẳng thức Vacs

Thêm một số bài tập về điểm Fermat, đẳng động, đường tròn Apllonius,..

Bài 1: Cho tam giác ABC và một điểm P. Dựng các đường tròn P − apollonius của các tam giác

APB, APC.BPC . Chứng minh rằng ba đường tròn này còn có một điểm chung P' khác P

Gợi ý: Gọi (N),(M),(L) là đường tròn P − apollonius của các tam giác APB, APC.BPC

Chú ý rằng BL là đường đối trung ngoài của tam giác BPC, lập tỉ số rồi sử dụng định lý Menelaus.

Bài 2: Cho A, B, C nằm trên đường tròn (K) sao cho tam giác ABC là tam giác nhọn và P là

điểm bên trong (K). Gọi X,Y, Z là giao điểm thứ hai của AP, BP, CP với (K). Xác định vị trí

của điểm P sao cho ∆XYZ đều

Gợi ý: Gọi F là điểm liên hợp đẳng giác với P của tam giác ABC thì ta có $\angle APC+ \angle AFC =180^o + \angle B$ và biến đổi góc được $\angle APC= 60^o + \angle B$.
Vậy F là điểm Fermat . P liên hợp đẳng giác với F chỉ có thể là điểm đẳng động J của tam giác ABC

Bài 3: Cho D là một điểm bên trong tam giác nhọn ABC sao cho AB = a.b, AC = a.c, AD =

a.d, BC = b.c, BD = b.d và CD = c.d. Chứng minh $\angle ABD+ \angle  ACD= 60^o$

Gợi ý:- D là điểm chung của 3 đường tròn apollonius của ∆ABC và chính là điểm đẳng động

thứ nhất của ∆ABC.
-Gọi tam giác đều thủy túc của điểm đẳng động D của ∆ABC.

Bài 4: Chứng minh đường tròn A − apollonius tạo với các đường tròn B − apollonius, C − apollonius

của ∆ABC một góc $120^o$

Bài 5: Cho tam giác ABC vuông tại A và H là chân đường cao vẽ từ A. Chứng minh các đường tròn

H − apollonius của các tam giác ∆AHB, ∆AHC giao nhau tại tâm đường tròn A − apollonius

của tam giác ABC
Bài 6: Cho tam giác ABC và phân giác trong BD, D ∈ AC. Đường thẳng BD cắt đường tròn (C)

ngoại tiếp ∆ABC tại điểm thứ hai là E. Đường tròn (ω) với đường kính DE cắt lại (C) tại F.

Chứng minh BF là đường đối trung của tam giác ABC

Bài 7: Cho tam giác đều XYZ nội tiếp trong đường tròn (O) và điểm P tùy ý trong tam giác

XYZ và không nằm trên các cạnh, PX, PY, PZ cắt (O) tại A, B, C theo thứ tự. Gọi D, E, F theo

thứ tự là tâm đường tròn nội tiếp các tam giác PBC, PCA, PAB. Chứng minh AD, BE, CF

đồng quy.

Mối liên hệ giữa điểm Fermat và điểm đẳng động

• Nhắc lại điểm Fermat :

Cho tam giác ABC. Dựng theo hướng ngoài 3 tam giác đều ABD, ACE, BCG. Ba đường

tròn (ABD),(ACE),(BCG) đồng quy tại điểm F gọi là điểm Fermat thứ nhất của tam giác

ABC

Nếu dựng theo hướng trong thì F gọi là điểm Fermat thứ hai của tam giác ABC


Khái niệm về hai điểm liên hợp đẳng giác: Cho điểm M bên trong tam giác ABC. Khi

đó các đường thẳng đối xứng của các đường thẳng AM.BM, CM qua tia phân giác đồng

quy tại M'

. Điểm M' được gọi là điểm liên hợp đẳng giác của điểm M trong tam giác ABC

Ngoài ra: $\angle BMC+ \angle BM'C=(180^o- \angle MBC -\angle MCB)+(180^o -\angle M'BC- \angle M'CB) $
$ =(180^o -\angle M'BA -\angle M'CA )+(180^o- \angle M'BC -\angle M'CB) \\ =360-\angle B- \angle C=180^o + \angle A$

Ta có định lý sau: Điểm đẳng động và điểm Fermat là hai điểm liên hợp đẳng giác

Gọi LMN là tam giác đều thủy túc của điểm đẳng động thứ nhất J. Do các tứ giác

JMAN, JNBL, JLCN nội tiếp được nên ta có : $\angle BJC= \angle BJL +\angle LJC =\angle BNL + \angle LMC =\\ 180 - ( \angle LNJ + \angle JNA ) +180^o - \angle LMJ -\angle JMA =180^o- ( \angle LNJ + \angle LMJ) \\ =60^o + \angle JNM + \angle JMN =60^o + \angle JAM + \angle JAN =\angle BAC +60^o$

Mà $ \angle BFC =120^o$ nên : $\angle BFC + \angle BJC =180 ^o + \angle A $

Chứng tỏ F liên hợp đẳng giác của J

Đường kính Brocard và tam giác đều " thủy túc "

Bài toán 1: Cho tam giác ABC nội tiếp (O), và K là điểm Lemoine  có J, J' là hai điểm đẳng động . Nếu OK cắt (O) tại Q và R thì (QRJJ')=-1

Chứng minh:

Ta đã chứng minh O,K, J, J' thẳng hàng và do J và J' nghịch đảo nhau đối với (O) nên (QRJJ')=-1

QR được gọi là đường kính Brocard của tam giác ABC

Bài toán 2:

Chứng minh rằng có đúng hai điểm đối với một tam giác sao cho chân đường vuông góc hạ

từ chúng đến ba cạnh của tam giác tạo thành một tam giác đều

Chứng minh:

Theo bài toán 1 điểm có hình chiếu lên 3 cạnh tam giác tạo thành 1 tam giác cân nằm

trên một đường tròn apollonius.
Để có tam gíac đều thì điểm đó phải nằm trên 3 đường tròn apollonius của tam giác tức

là hai điểm đẳng động J, J'

của tam giác

Chú ý : Tam giác đều tạo bởi 3 hình chiếu của điểm J lên ba cạnh của tam giác thường

gọi là tam giác đều " thủy túc " của điểm J

Bài toán 3: Trong tất cả các tam giác đều có đỉnh nằm trên ba cạnh của một tam giác thì tam giác đều

thủy túc của điểm đẳng động thứ nhất J của tam giác có diện tích nhỏ nhất.

Chứng minh:

Nhắc lại định lý Miquel :

Cho tam giác ABC và ba điểm L, M, N nằm trên BC, CA, AB. Khi đó ba đường tròn

(AMN),(BLN),(CLM) đồng quy



Gọi các điểm như trong hình vẽ
Ta có:

$\widehat{JLM}=\widehat{JCM}=\widehat{JL'M} \\ \widehat{JLN}=\widehat{JBN}=\widehat{JL'N'} $

Cộng lại ta được:

$\widehat{MLN}=\widehat{M'L'N'}=60^o$

Phép đồng dạng ( vị tự quay) tâm J với tỷ số $r=\frac{JL'}{JL} \le 1, \alpha=\widehat{LJL'}$

biến tam giác

LMN thành tam giác L'M'N'. Theo bài toán 3 suy ra J là điểm đẳng động thứ nhất.

Điểm đẳng động

Ta tiếp theo chuỗi bài tập về đường tròn Apollonius 
Bài 3: Ba đường tròn Apollonius của một tam giác có hai điểm chung. Ta gọi hai điểm chung này là hai điểm đẳng động (Isodynamic Point)
Lời giải
Giả sử J thuộc đường tròn Apollonius ứng với điểm A, và B của tam giác ABC thì ta có:

$\frac{JB}{JC}=\frac{AB}{AC},\frac{JC}{JA}=\frac{BC}{BA} \\ \Rightarrow \frac{JB}{JA}=\frac{CB}{CA}$ Vậy J thuộc đường tròn Apllonius ứng với điểm C

Tương tự J' ở ngoài (O).

Bài 4: Tâm của ba đường tròn này thẳng hàng.

Suy ra từ bài toán 3 nên ba đường tròn đồng trục.

Bài 5: Cho O và K là tâm đường tròn ngoại tiếp và điểm Lemoine của tam giác ABC, J, J' là hai điểm đẳng động . Chứng minh :
a) J, J', K, O thẳng hàng.
b) J' là nghịch đảo của J đối với đường tròn nghịch đảo (O)

Lời giải:
a)
Ta cần bổ đề sau:

Bổ đề 1: Nếu đường tròn (O,r) trực giao với hai đường tròn (A, p),(B, q) thì O thuộc trục đẳng phương của (A, p).(B.q).

Chỉ cần chứng minh phương tích điểm O đối với hai đường này bằng nhau ( bằng $r^2$)
Ta có:
Đường tròn ngoại tiếp tam giác ABC trực giao với các đường tròn Apollonius và

các đường tròn Apollonius có trục đẳng phương là JJ' nên O thuộc JJ'

ta có L, M, N (L, M, N lần lượt là tâm đường tròn Apollonius góc A, góc B, góc C) là cực của các đối cực đối trung của tam giác ABC đối

với đường tròn nghịch đảo (O). Điểm Lemoine K là giao điểm của 3 đường đối trung nên

đối cực của K đối với đường tròn nghịch đảo (O) qua L, M, N ( theo định lý La Hire )
Ta có cực K của đối cực LMN thuộc đường thẳng qua O và vuông góc với LMN và đường

thẳng này là trục đẳng phương OJJ'

Vậy ta đã chứng minh xong câu a)

b) Dùng tính chất (O) và (O') trực giao thì 1) (O') bất biến trong phép nghịch đảo với đường tròn nghịch đảo (O) ( vì phương tích

của O' đối với (O) bằng phương số nghịch đảo )

2) (O) bất biến trong phép nghịch đảo với đường tròn nghịch đảo (O')

Ta có (O),(L) ( (L) là A − apollonius ) trực giao và O, J, J', thẳng hàng và J, J' thuộc (L) nên J, J' là nghịch đảo nhau trong phép nghịch đảo với đường tròn nghịch đảo

Tổng hợp hình vẽ:

bấm vào để thấy rõ hơn.

Dùng đường tròn Apollonius vào bài toán tìm tập hợp điểm

Lưu ý với một số bài toán dùng đường tròn $Apollonius$ để tìm tập hợp điểm, ta phải xét tỉ số bằng 1 là thiết yếu do khi đó quỹ tích sẽ là đường trung trực của đoạn thẳng chứ không còn là đường tròn nữa. Chúng ta xét các bài tập sau:
Bài 1: Cho tam giác ABC và điểm P . Gọi $P_1, P_2, P_3$ là hình chiếu vuông góc của P trên BC, CA, AB

theo thứ tự. Tìm quỹ tích của P sao cho $P_1P_2 = P_1P_3$

Lời giải
Ta có tứ giác BP1PP3 nội tiếp trong đường tròn đường kính BP nên theo định lý sin ta có

$P_1P_3 = BP sin B$

Tương tự

$P_1P_2 = CP sin C$

$\Rightarrow P_1P_2=P_1P_3 \Leftrightarrow  \frac{BP}{CP}=\frac{sinC}{sinB}=\frac{AB}{AC}$
Vậy quỹ tích P là đường tròn A − apollonius của tam giác ABC
Nếu AB=AC thì P thuộc trung trực BC.
Bài 2 (VMO 1999): Cho tam giác ABC nội tiếp (O). Hãy xác định vị trí của P không thuộc đường tròn để các đường PA, PB, PC cắt lại đường tròn ở A', B', C' sao cho tam giác A'B'C' là vuông cân với đáy B'C'.

Lời giải:

Hình vẽ chỉ mang tính chất minh họa vì lời giải sử dụng góc định hướng nên không cần xét hai trường hợp P ở trong và ngoài đường tròn
Giả sử điểm P thỏa mãn đề bài, không mất tính tổng quát giả sử (AB, AC) là góc dương. P phải thỏa hai điều kiện sau:
Điều kiện 1: (PB,PC)=(PB,AB)+(AB,AC)+(AC,PC)
=(BP,BA)+(CA,CP)+(AB,AC).

$\Rightarrow (PB,PC)\equiv (A'B',A'A)+(A'A,A'C')+(AB,AC)\\ \equiv (A'B',A'C')+(AB,AC) \equiv \frac{\pi}{2}+(AB,AC)(mod \pi)$


Nếu tam giác ABC vuông tại A thì P thuộc BC.

Nếu tam giác ABC không vuông tại A thì P nằm trên đường tròn $C$ đi qua B, C và chắn cung $\frac{\pi}{2}+(AB,AC)$

Điều kiện 2: Ta có:

$\triangle ABP \sim \triangle B'A'P \Rightarrow \frac{AB}{A'B'}=\frac{PB}{PA'}\\\frac{AC}{A'C'}=\frac{CP}{A'P}\\B'A'=A'C'\Rightarrow \frac{PB}{PC}=\frac{AB}{AC} (3)$

Nếu AB=AC thì P thuộc trung trực BC
Nếu AB khác AC thì P thuộc đường tròn Apollonius $(C_p)$ thỏa mãn (3)

Kết hợp những điều kiện trên suy ra được tập hợp điểm.

Bài 3: (VMO 2000): Trên mặt phẳng cho trước $(O_1)$ tâm $O_1$ bán kính $r_1$ và $(O_2)$ tâm $O_2$ bán kính $r_2$. Trên đường tròn $(O_1)$ lấy một điểm $M_1$, trên đường tròn $(O_2)$ lấy một điểm $M_2$ sao cho đường thẳng $O_1M_1$ cắt $O_2M_2$ tại một điểm Q. Cho $M_1$ chuyển động trên đường tròn $(O_1)$, $M_2$ chuyên động trên đường tròn $(O_2)$ cùng theo chiều kim đồng hồ và với vận tốc góc như nhau.
1) Tìm quỹ tích trung điểm đoạn thẳng $M_1M_2$

2) Chứng minh rằng đường tròn ngoại tiếp tam giác $M_1QM_2$ luôn đi qua một điểm cố định.

Lời giải:

Do điều kiện bài toán ta sẽ dùng góc định hướng để lời giải ngắn gọn và chuẩn xác.

Câu 1  bạn đọc dùng Vecto rồi giải, ta chỉ quan tâm đến câu 2 vì có liên quan đến đường tròn Apollonius

Gọi P là giao điểm thứ 2 của $(M_1QM_2)$ và $O_1QO_2$ thì:

$\left ( \overrightarrow{PM_1}, \overrightarrow{PO_1} \right )=\left ( \overrightarrow{PM_1}, \overrightarrow{PM_2} \right )+\left ( \overrightarrow{PM_2}, \overrightarrow{PO_2} \right )+\left ( \overrightarrow{PO_2}, \overrightarrow{PO_1} \right )=\left ( \overrightarrow{PM_2}, \overrightarrow{PO_2} \right )\\ \left ( \overrightarrow{O_1M_1},\overrightarrow{O_1P} \right )=\left ( \overrightarrow{O_1M_1},\overrightarrow{O_2M_2} \right )+\left ( \overrightarrow{O_2M_2},\overrightarrow{O_2P} \right )+\left ( \overrightarrow{O_2P},\overrightarrow{O_1P} \right )=\left ( \overrightarrow{O_2M_2},\overrightarrow{O_2P} \right )$

do đó tam giác $PO_1M_1 \sim PO_2M_2$. Suy ra $\frac{PO_1}{PO_2}=\frac{r_1}{r_2}$ Do đó nếu $r_1=r_2$ thì P thuộc trung trực $O_1O_2$ còn nếu không thì P thuộc đường tròn Apollonius dựng trên đoạn $O_1O_2$ cố định, theo tỉ số $\frac{r_1}{r_2}$ không đổi.

Mặt khác: $\left ( \overrightarrow{PO_1}, \overrightarrow{PO_2} \right )=\left ( \overrightarrow{QO_1}, \overrightarrow{QO_2} \right )=\alpha$ (Không đổi)

Kết hợp những điều trên suy ra P cố định.

Nhận xét: Để ý rằng Q là tâm của phép vị tự quay biến tam giác $PM_1M_2$ thành tam giác $PO_1O_2$

Thứ Hai, 24 tháng 10, 2016

Một số hàm số học và ứng dụng

I) Hàm phần nguyên:
1) Định nghĩa
Phần nguyên của một số thực x là số nguyên lớn nhất không vượt quá x. Kí hiệu là [x].

2) Tính chất:

a) nếu x>y thì [x] $\ge$ [y]

b) [n+x]=n+[x] (với n nguyên)

c) [x+y] $\ge $ [x]+[y]

d)[2x]+[2y] $ \ge $ [x]+[y]+[x+y]

e) [([x]/d)]=[x/d]

f) Cho x là một số thực dương và d là một số nguyên dương. Khi đó các số nguyên dương là bội của d không vượt qua x là [x/d].

Từ đây ta có định lý:
Trong phân tích tiêu chuẩn của: $n!=p_1^{a_1}p_2^{a_2}..p_k^{a_k}$ số mũ $a_i$ của $p_i$ được tính theo công thức:

$a_i=\left [ \frac{n}{p_i} \right ]+\left [ \frac{n}{p_i^2} \right ]+\left [ \frac{n}{p_i^3} \right ]...$

3) Ứng dụng:

Bài 1: Chứng minh rằng với mọi số nguyên dương m,n ta có (2m)!(2n)! chia hết cho m!n!(m+n)!.

Lời giải:

Gọi p là một số nguyên tố bất kì. Theo định lí thì số mũ của p trong phân tích tiêu chuẩn của m!n!(m+n)! là:
$a=\sum_{k=1}^{\infty }\left [ \frac{m}{p^k} \right ]+\left [ \frac{n}{p^k} \right ]+\left [ \frac{m+n}{p^k} \right ]$

số mũ của p trong phân tích tiêu chuẩn của (2m)!(2n)! là:

$b=\sum_{k=1}^{\infty }\left [ \frac{2m}{p^k} \right ]+\left [ \frac{2n}{p^k} \right ]$

Theo tính chất d) ta có:

$\left [ \frac{m}{p^k} \right ]+\left [ \frac{n}{p^k} \right ]+\left [ \frac{m+n}{p^k} \right ] \le \left [ \frac{2m}{p^k} \right ]+\left [ \frac{2n}{p^k} \right ]$
Từ đó $b \ge a$. Vậy ta có đpcm.
Bài 2:

Cho số nguyên dương $a, n$ sao cho tất cả các ước nguyên tố của $a$ đều lớn hơn $n.$ Chứng minh rằng $(a-1)(a^{2}-1)...(a^{n-1}-1)$ chia hết cho $n!.$

Cách 1:



Gọi $p$ là 1 ước nguyên tố của $n$.

Có $a^{p-1}\equiv 1(modp)$ nên $V_{p}(VT)\geq V_{p}(\prod_{i=1}^{[\frac{n}{p-1}]}(a^{(p-1)i}-1))\geq [\frac{n}{p-1}] \\ V_{p}(n!)=[n/p]+[n/p^{2}]+...[n/p^{s}]\leq n/p+n/p^{2}+...+n/p^{s}=\frac{n-n/p^{s}}{p-1}\leq [\frac{n}{p-1}] \\ --> Q.E.D.\blacksquare$

Cách 2
Bổ đề : Cho $p$ là một số nguyên tố và $n$ là số nguyên dương . Kí hiệu $s_p(n)$ là tổng các chữ số của $n$ viết trong hệ cơ số $p$
Khi đó $v_p(n!)=\frac{n-s_p(n)}{p-1}$
Chứng minh bổ đề : Đặt $n=a_kp^k+a_{k-1}p^{k-1}+..+a_1p+a_0,a_i \in \{1,2,..,p-1\},i=\overline{1,k}$
Theo định lí Legendre : $v_p(n!)=\sum_{i=0}^k [\frac{n}{p_i}]=a_k\frac{p^k-1}{p-1}+a_{k-1}\frac{p^{k-1}-1}{p-1}+a_1$
$=\frac{(a_kp^k+a_{k-1}p^{k-1}+..+a_0)-(a_k+a_{k-1}+..+a_0}{p-1}=\frac{n-s_p(n)}{p-1}$ (đpcm)
Đi vào bài toán
Vì tất cả các ước nguyên tố của $a$ đều lớn hơn $n$ nên $(a,n)=1$ . Giả sử $p$ là ước nguyên tố của $n!$ thì ta có $(a,p)=1$.
Theo định lí Fermat ta có $a^{k(p-1)} \equiv 1 \pmod{p},k \ge 1$ . Mặt khác ta có $v_p(n!)=\frac{n-s_p(n)}{p-1} \le [\frac{n-1}{p-1}]$
Ta có
$v_p(\prod_{k=1}^{n-1}(a^k-1))=\sum_{k=1}^{n-1}v_p(a^k-1) \ge \sum_{k=1}^{k(n-1) \le n} v_p(a^{k(p-1)}-1) \ge [\frac{n}{p-1}] \ge [\frac{n-1}{p-1}] \ge v_p(n!)$
Vậy $n!|\prod_{k=1}^{n-1}(a^k-1)$
II) Hàm số các ước của một số tự nhiên.

1) Định nghĩa: Cho số nguyên dương n. Kí hiệu d(n) là số các ước của n

2) Định lí:
a) Giả sử $n=p_1^{a_1}p_2^{a_2}..p_k^{a_k}$ là phân tích tiêu chuẩn của n>1.
Khi đó $d(n)=(a_1+1)(a_2+1)..(a_k+1)$

b) Với mọi n ta có bất đẳng thức $d(n) < 2 \sqrt{n}$

III) Hàm tổng các ước

1) Định nghĩa: Cho số nguyên dương n. Ta kí hiệu $\sigma (n)$ là tổng các ước của n.

2) Định lí:
a) Hàm số $\sigma (n)$ có tính chất nhân tính. Nghĩa là nếu a,b là hai số nguyên tố cùng nhau thì:
$\sigma (ab)=\sigma (a).\sigma (b)$

b) Giả sử $n=p_1^{a_1}p_2^{a_2}..p_k^{a_k}$ là phân tích tiêu chuẩn của n>1. Khi đó:

$\sigma (n)=\left (\frac{p_1^{a_1+1}-1}{p_1-1}  \right )\left (\frac{p_2^{a_2+1}-1}{p_2-1}  \right )..\left (\frac{p_k^{a_k+1}-1}{p_k-1}  \right )$

c) n là số nguyên tố khi và chỉ khi $\sigma (n)=n+1$.

d) $\sigma (n)$ là một số lẻ nếu và chỉ nếu n là số chính phương hoặc $\frac{n}{2}$ là số chính phương.

Liên quan đến hàm $\sigma (n)$ ta có số hoàn chỉnh.

Định lý về số hoàn chỉnh:

e) Nếu k là số tự nhiên sao cho $2^k-1$ là một số nguyên tốt thì số $n=2^{k-1}(2^k-1)$ là một số hoàn chỉnh.

f)Nếu n là một số hoàn chỉnh chẵn thì n có dạng:
$n=2^k.(2^{k+1}-1)$

IV) Hàm số Euler:

1) Định nghĩa: 
Cho số tự nhiên $n \ge 1$. ta kí hiệu $\varphi (n)$ là  số các số tự nhiên bé hơn n và nguyên tố cùng nhau với n.

2) Định lý:

Hàm $\varphi (n)$ có tính chất nhân tính theo nghĩa: Nếu a,b là hai số nguyên tố cùng nhau thì:

$\varphi (ab)=\varphi (a)\varphi (b)$

Chú ý rằng những định lý và các khái niệm đều được dùng thẳng trong kì thi VMO ( Theo công văn số 1447/ KTKĐCLGD-KT ngày 27/10/2015). Một điều lạ là kì thi VMO 2016 cho chứng minh định lí e) và f).

Chủ Nhật, 23 tháng 10, 2016

IMO shortlist 2002 và những vấn đề liên quan.

Đề bài: Đường tròn nội tiếp (I) của tam giác ABC tiếp xúc với BC tại K. Gọi M là trung điểm đường cao AD. KM cắt (I) tại N. Chứng minh rằng (BCN) tiếp xúc với (I) tại N.

Lời giải:

Đặt $S=KA\cap \Omega$,và đặt $T$ là điểm đối xứng $K$ qua I . Gọi $X,Y$ là các tiếp điểm của (I) trên $CA,AB$.

$AD$ // $KT$ nên $(KT,KN;KS,KD)=-1$. suy ra $KTSN$ điều hòa, vậy tiếp tuyến với $(I)$ tại $K,S$ đồng quy trêb $NT$. Mặt khác, tiếp tuyến tại $X,Y$ đồng quy $KS$, nên $KXSY$ là tứ giác điều hòa, Nghĩa là tiếp tuyến tại $K,S$ đồng quy $XY$.

Từ đây suy ra $BC,XY,TN$ Đồng quy. Gọi $P=XY\cap BC$, suy ra $(B,C;K,P)=-1$,và vì $\angle KNP=\angle KNT=\frac{\pi}2$, nên $NK$ là phân giác của $\angle BNC$.

Gọi $B'=NB\cap \Omega,\ C'=NC\cap \Omega$, Ta có:
$BB^{\prime} \cdot BN = BK^2$
$CC^{\prime} \cdot CN = CK^2$. Suy ra:
$\frac{BB^{\prime} \cdot BN}{CC^{\prime} \cdot CN} = \frac{BK^2}{CK^2}$
Nhưng NK là phân giác của góc BNC nên:
$\frac{BK}{CK} = \frac{BN}{CN}$
Vì vậy:
$\frac{BB^{\prime} \cdot BN}{CC^{\prime} \cdot CN} = \frac{BN^2}{CN^2}$
Suy ra:
$\frac{BB^{\prime}}{CC^{\prime}} = \frac{BN}{CN}$
Nên:
 $B'C'\|BC$, nên $N$ là tâm vị tự biến $(I)$ thành $(BNC)$ Hay (I) tiếp xúc (BNC) tại N vậy ta có điều phải chứng minh.

Ta có thêm những kết quả sau:

-NK đi qua tâm bàng tiếp góc A. (Gọi M là trung điểm BC và lưu ý AT đi qua tiếp điểm bàng tiếp trên BC Dùng hàng điểm để chứng minh)

-Nếu gọi $N_B$ tương tự như điểm N ứng với góc B, $N_C$ ứng với góc C thì $NK, N_BX, N_CY$ đồng quy tại S thuộc OI( Dùng cực đối cực)

-  Trục đẳng phương của $N_BAC, N_CAB$ và $N_BX, N_CY$ đồng quy tại một điểm thuộc OI. ( ý đầu dùng định lý 4 điểm, ý sau có thể dùng tâm vị tự, cực đối cực) suy ra điểm đồng quy này là S.

Ngoài ra rất nhiều bài toán đẹp có thể được phát triển từ những mô hình này.

Một bài toán lạ !

Bài toán (Hong Kong TST 2017); Tìm chữ số thập phân đầu tiên sau dấu phẩy của số: $\displaystyle \frac1{1009}+\frac1{1010}+\cdots + \frac1{2016}$

Lời giải:

Đặt $a_n=\frac{1}{n+1}+\frac{1}{n+2}+\ldots+\frac{1}{2n}$ và chú ý rằng $a_n-a_{n-1}=\frac{1}{2n(2n-1)}$, vì vậy $a_n=\sum_{k=1}^n\frac{1}{2k(2k-1)}<\sum_{k=1}^\infty\frac{1}{2k(2k-1)}=\sum_{k=1}^{\infty}\frac{(-1)^{k+1}}{k}=\ln 2<0.7$. Và $a_{1008}=\sum_{k=1}^{1008}\frac{1}{2k(2k-1)}>\frac{1}{2}+\frac{1}{12}+\frac{1}{30}>0.6$, Vậy đáp số là 6.


Thứ Bảy, 22 tháng 10, 2016

Chứng minh tồn tại vô hạn trong số học

Đề bài: Cho m là một số nguyên dương. Chứng minh rằng tồn tại vô số nguyên dương n sao cho $m|3.2^n+n$

Lời giải:

Ta sẽ chứng minh quy nạp theo $m$, rõ ràng $m=1,2,3,4$ là hiển nhiên
Giả sử khẳng định đúng với mọi $m\leq t$ với số nguyên $t>3$
Đặt $d=ord_{t+1}{2}$ và $e=gcd(d,t+1)$, rõ ràng $e\leq d \leq \phi (t+1) \leq t$
Theo nguyên lí quy nạp phải tồn tại vô hạn số nguyên $a$ sao cho $e\mid 3\times 2^a+a$
Đặt $3\times 2^a+a=ef$ ( $f\in \mathbb{Z}^+$ )
Với mọi $g\in \mathbb{Z}^+$ Ta có $3\times 2^{a+dg}+(a+dg) \equiv 3  \times 2^a+a+dg =ef+dg (mod t+1)$
Ta cần chứng minh rằng tồn tại $g\in \mathbb{Z}^+$ sao cho $ef+dg\equiv 0 (mod t+1)$
Tương đương với $gcd(d,t+1) \mid -ef\Leftrightarrow e\mid -ef$ (luôn đúng) 9 (điều kiện cần và đủ của phương trình đồng dư)
vậy ta đã chứng minh tồn tại $n_1=a+dg\in \mathbb{Z}^+$ sao cho $t+1 \mid 3\times 2^{n_1}+{n_1}$
Nhưng vì tồn tại vô số nguyên dương a $a$, ta lấy $a$ lớn hơn $n_1$ ta lại có một số nguyên dương khác thỏa mãn đề bài,
Điều này có nghĩa là tồn tại vô số $n\in \mathbb{Z}^+$ sao cho $t+1\mid 3\times 2^n+n$ Điều phải chứng minh.


Thứ Năm, 20 tháng 10, 2016

Bổ đề về hàm liên tục ứng dụng vào giải phương trình hàm

Ta có bổ đề sau:

Nếu f: R-> R vừa là hàm đơn ánh, vừa là hàm liên tục thì f đơn điệu.

Chứng minh:

Vì f là đơn ánh, ta chứng minh nếu tồn tại x<y sao cho f(x)< f(y) thì f đồng biến, (nếu với mọi x<y mà f(x) > f(y) thì hiển nhiên f nghịch biến). Giả sử f không đồng biến sẽ có 3 trường hợp sau xảy ra:

1) z<x<y và f(z) >f(x), f(x) <f(y)
2) x<y<z và f(z) < f(y), f(x) < f(y),
3) x<z<y và (f(z)-f(x))(f(z)-f(y)) >0

Ta sẽ chứng minh 1) sai, 2) tương tự, 3 sai suy ra từ 1) và 2) sai.

Chọn M sao cho f(x) <M < min {f(y), f(z)}. Theo tính chất hàm liên tục vì z<x<y nên tồn tại a sao cho z<a<x và f(a) =M, đồng thời tồn tại b sao cho x<b<y và f(b)=M. Như vậy a=b vì f đơn ánh nhưng điều này không thể xảy ra. Vậy f đơn điệu.

Như vậy ta có bài toán sau:

Tìm tất cả các hàm liên tục f: R-> thỏa mãn:

$f_{2017}(x)=x \forall x \in R$ (kí hiệu f_n(x)=f(f..f(x)..) n lần f.
Lời giải
Ta có f là đơn ánh
Ta chứng minh f là hàm đồng biến
Giả sử f nghịch biến , ta có x<y thì f(x) > f(y).
Suy ra $f_2(x) <f_2(y)$ cứ tiếp tục như thế ta sẽ có x>y mâu thuãn.

Vậy f là đồng biến.
Nếu f(x)>x khi đó bằng quy nạp suy ra được $f_{2017}(x) >x$ tương tự f(x) <x.

Vậy$ f(x)=x (\forall x \in R)$


Thứ Tư, 19 tháng 10, 2016

Dùng định lý Pascal suy biến vào bài toán chia đôi

Ta có định lý Pascal đầy đủ cho lục giác, định lý Pascal suy biến là khi một số các đỉnh trùng nhau.

Ta xét bài toán sau:

Cho tam giác ABC, nội tiếp (O), ngoại tiếp (I). BI cắt AC, (O) lần lượt tại $B_0, B_1$. Tương tự $C_0, C_1$ . Gọi S là giao điểm của $C_0B_1 và B_0C_1$. Chứng minh rằng SI chia đôi BC.

Lời giải:

Đặt $ T \equiv B_0C_0 \cap B_1C_1 $ và $ X \equiv AI \cap BC, Y \equiv AT \cap BC $ .

Áp dụng định lý Pascal suy biến cho lục giác $ AABB_1C_1C$ $ \Longrightarrow AT $ là tiếp tuyến của $ \odot (ABC) $ ,
Để ý rằng $ B_1C_1 $ là trung trực $ AI $ nên $ TA=TI $ . ... $ (\star) $

Ta có: $ YA=YX \Longrightarrow $ Kết hợp với $ (\star) $ Ta có $ TI \parallel XY \equiv BC $ ,
Vì thế từ tứ giác $B_0C_1B_1C_1$ toàn phần suy ra $ I(B,C;T,S)=-1 \Longrightarrow IS $ đi qua trung điểm $ BC $ .

Thứ Bảy, 15 tháng 10, 2016

Đại số hóa bất đẳng thức lượng giác

Có nhiều bài toán bất đẳng thức ở dạng tam giác lượng thì việc chứng minh rất dễ dàng . Xong khi nó bị mã hóa về dạng đại số thì sẽ khá gây khó dễ để chứng minh nếu ta không hình dung được bản chất lượng giác ban đầu của nó
1.1 Biểu diễn các đại lượng theo biến mới
Cho tam giác ABC có độ dài các cạnh là {a,b,c }.Đặt p=\frac{a+b+c}{2}{a=\frac{y+z}{2}},  b=\frac{z+x}{2},  c=\frac{x+y}{2}  hay {x=2\left ( p-a \right ), y=2\left ( p-b \right ), z=2\left ( p-c \right )}. Ta có các biểu diễn sau
{sinA=\frac{2\sqrt{xyz\left ( x+y+z \right )}}{\left ( x+y \right )\left ( x+z \right )}},  {sinB=\frac{2\sqrt{xyz\left ( x+y+z \right )}}{\left ( y+x \right )\left ( y+z \right )}},  {sinC=\frac{2\sqrt{xyz\left ( x+y+z \right )}}{\left ( z+x \right )\left ( z+y \right )}},
{cosA=\frac{x\left ( x+y+z \right )-yz}{\left ( x+y \right )\left ( x+z \right )}},   {cosB=\frac{y\left ( x+y+z \right )-zx}{\left ( x+y \right )\left ( y+z \right )}},   {cosC=\frac{z\left ( x+y+z \right )-yx}{\left ( x+z \right )\left ( y+z \right )}}
{tanA=\frac{2\sqrt{xyz\left ( x+y+z \right )}}{x\left ( x+y+z \right )-yz}},  {tanB=\frac{2\sqrt{xyz\left ( x+y+z \right )}}{y\left ( x+y+z \right )-zx}},  {tanC=\frac{2\sqrt{xyz\left ( x+y+z \right )}}{z\left ( x+y+z \right )-xy}}
{cotA=\frac{x\left ( x+y+z \right )-yz}{2\sqrt{xyz\left ( x+y+z \right )}}},  {cotB=\frac{y\left ( x+y+z \right )-zx}{2\sqrt{xyz\left ( x+y+z \right )}}},  {cotC=\frac{z\left ( x+y+z \right )-xy}{2\sqrt{xyz\left ( x+y+z \right )}}}
sin\frac{A}{2}=\sqrt{\frac{yz}{\left ( x+y \right )\left ( x+z \right )}},  sin\frac{B}{2}=\sqrt{\frac{zx}{\left ( x+y \right )\left ( y+z \right )}},  sin\frac{C}{2}=\sqrt{\frac{xy}{\left ( z+y \right )\left ( x+z \right )}}
cos\frac{A}{2}=\sqrt{\frac{x\left ( x+y+z \right )}{\left ( x+y \right )\left ( x+z \right )}},  cos\frac{B}{2}=\sqrt{\frac{y\left ( x+y+z \right )}{\left ( x+y \right )\left ( y+z \right )}},  cos\frac{C}{2}=\sqrt{\frac{z\left ( x+y+z \right )}{\left ( x+z \right )\left ( y+z \right )}}
1.2 Xây dựng các bất đẳng thức đại số 
Ví dụ  1. Cho các số thực {x, y, z}  dương. Chứng minh rằng {\sum_{cyclic}\frac{x}{\left ( x+y \right )\left ( x+z \right )}\geq \frac{9}{4\left ( x+y+z \right )}}
HD: bất đẳng thức này được sinh ra từ bất đẳng thức {cosA+cosB+cosC\leqslant \frac{3}{2}}
Ví dụ  2. [Văn Đức Chín] Cho các số thực {x, y, z}  dương có tổng bằng 1. Chứng minh rằng
{\sum \sqrt{x\left ( 1-x \right )}\geq \frac{4\sqrt{xyz}}{\sqrt{\left ( 1-x \right )\left ( 1-y \right )\left ( 1-z \right )}}}
HD: bất đẳng thức này được sinh ra từ bất đẳng thức {sinA+sinB+sinC\leq cos\frac{A}{2}+cos\frac{B}{2}+cos\frac{C}{2}}
1.3. Bài tập
Bài 1. [Văn Đức Chín] Cho các số thực {x, y, z}  dương . Chứng minh rằng
{x^2+y^2+z^2+xy+yz+zx\geq 2\sqrt{3}\sqrt{xyz\left ( x+y+z \right )}}
Bài 2. [IMO,1983] Cho các số thực {x, y, z}  dương . Chứng minh rằng
{x^{3}y+y^{3}z+z^{3}x\geq x^{2}y^{2}+y^{2}z^{2}+z^{2}x^{2}}
Bài 3. [Văn Đức Chín] Cho các số thực {x, y, z}  dương . Chứng minh rằng
{1+\frac{4xyz}{\left ( x+y \right )\left ( y+z \right )\left ( z+x \right )}\leq \sum_{cyclic}\sqrt{\frac{yz}{\left ( x+y \right )\left ( x+z \right )}}}
Bài 4. [Văn Đức Chín] Cho các số thực {x, y, z}  dương . Chứng minh rằng
{64xyz\left ( x+y+z \right )^3\leq 27\left ( x+y \right )^2\left ( y+z \right )^2\left ( z+x \right )^2}
Bài 5. [Văn Đức Chín]Cho các số thực {x, y, z}  dương thỏa mãn {x+y+z> Max\left \{ \frac{xy}{z},\frac{yz}{x}, \frac{zx}{y} \right \}}. Chứng minh rằng {\frac{1}{x}+\frac{1}{y}+\frac{1}{z}+\frac{1}{x+y+z} \geq 10\sqrt{\frac{x+y+z}{3xyz}}}
Bài 6. [MO Korea 1998] Cho các số thực {x, y, z}  dương thỏa mãn {xyz=x+y+z}. Chứng minh rằng
{\frac{1}{\sqrt{1+x^2}}+\frac{1}{\sqrt{1+y^2}}+\frac{1}{\sqrt{1+z^2}}\leq \frac{3}{2}}
Bài 7. [Văn Đức Chín] Cho các số thực {x, y, z,{\alpha} }  dương , thỏa mãn {xy+yz+zx=1}. Chứng minh rằng
{x^{\alpha} +y^{\alpha} +z^{\alpha}\geq 3^{1-\frac{{\alpha} }{2}}}
Bài 8. [Văn Đức Chín] Cho các số thực {x, y, z}  dương có tổng bằng 1 và thỏa mãn {1> Max\left \{ \frac{xy}{z},\frac{yz}{x}, \frac{zx}{y} \right \}}. Chứng minh rằng
{\sqrt{\frac{1}{xyz}}+\sqrt{\left ( 1+x \right )\left ( 1+y \right )\left ( 1+z \right )}\geq \frac{7\sqrt{3}}{144}}

Bất đẳng thức tuyển sinh lớp 10 chọn lọc

Trong bài viết này, tác giả giới thiệu một số bài BĐT nhẹ nhàng nhưng ý tưởng tương đối mới, mức độ phù hợp với đề thi tuyển sinh vào lớp...